LSAT and Law School Admissions Forum

Get expert LSAT preparation and law school admissions advice from PowerScore Test Preparation.

 fersian
  • Posts: 19
  • Joined: Jan 19, 2019
|
#66472
Hi Steve,
Is it possible that 'C' is also classified as a mistaken reversal?

I completely see why 'A' is correct, but when I was down to my contenders, this is what I noticed:

The conclusion can be diagrammed as :
want to increase size of applicant pool --> raise tuition and fees
contrapositive: ~raise tuition and fees ---> ~increase size of applicant pool

For choice 'C', I interpreted it to say: raise in tuition and fees --> larger applicant pool

Is my line of thinking correct on this one?
 James Finch
PowerScore Staff
  • PowerScore Staff
  • Posts: 943
  • Joined: Sep 06, 2017
|
#66763
Hi fersian,

Yes, (C) is a Mistaken Reversal of the argument's conclusion, and not an assumption required by it. Conditional conclusions like the one here can be tricky, but the Assumption Negation technique can quickly eliminate an answer choice like (C):

An increase in tuition would not guarantee a larger applicant pool :arrow: if we want a larger applicant pool, we don't need to raise tuition

Just because a condition isn't sufficient to ensure another condition occurring, doesn't mean it isn't necessary to that condition. Consider another example: betting money on a boxing match doesn't ensure that you'll win your bet, but in order to win a bet you do have to bet money.

(A) takes a different approach and clarifies that the causal explanation given in the stimulus, which the stimulus itself says is possible, not certain, is in fact the actual causal relationship. This is necessary, as without knowing this we can't have a 100% certain conditional conclusion.

Hope this clears things up!
 Imcuffy
  • Posts: 17
  • Joined: Aug 19, 2020
|
#81049
Hi,

I am having problems understanding why answer choice B is incorrect. The conclusion of his statement is: So, if we want to increase the size of our applicant pool, we need to raise our tuition and fees. His premise to back that up was: Prospective students and their parents conclude that the quality of education they would receive at this institution is not as high as that offered by institutions with higher tuition.

It seems to me that he is making a connection and assumption that because their tuition and fees are lower, that people believe they will receive a lower quality of education as well. It seemed that answer B replicated my thoughts.

Answer choice B: "the quality of a university education is dependent on the amount of tuition charged by the university"

Please help me to see if I am thinking about this the wrong way.

Thanks
 Adam Tyson
PowerScore Staff
  • PowerScore Staff
  • Posts: 5153
  • Joined: Apr 14, 2011
|
#81222
What your analysis is missing, Imcuffy, is the difference between people thinking that the quality of education is not as high and it actually being not as high. The argument is based in part on what the university president thinks parents and students will believe, but that belief could easily be false. The president might be thinking "we provide the highest quality education here, but our low tuition is creating a false perception, so we need to raise fees to counter that perception." The argument does not require assuming that the perception is valid! It does require the assumption that at least some people's perception of the quality of education at a university is influenced by the cost of tuition and fees, regardless of whether that is true or not.

Perception vs. reality, belief vs. truth, motives vs. actions, intentions vs. results - the LSAT loves to test us on the differences between these ideas, and tries to trick us into thinking that information about one item in a pair tells us something about the other item in that pair. Beware of those differences and the tricks that are based on them!
User avatar
 cgs174
  • Posts: 11
  • Joined: Jan 01, 2022
|
#93890
Hi!! When I looked back through this question A was an obvious answer but I was led astray with a wrong pre-phrase and quickly moved past A during the test.

Reading the stimulus the thing that stood out most to be was the flaw that it moves from considering charging too little tuition as one of potentially many possible explanations for low applications to concluding that if the school wants applications to increase, they must raise tuition. I saw the argument as making the conclusion that tuition increase was necessary to having more applicants when it was previously only asserted to be sufficient. I was preparing on that basis for a Flaw question but when I saw it was an assumption question I was then looking for an assumption that responded to this flaw. D) matched this pre-phrase very well. If there are no other reasons why applications are low, then the use of tuition increase as a necessary condition for higher applications is warranted.

I did think it odd that this question was in conflict with itself and probably should've realized on that basis that my analysis was leading me astray.

How can I avoid these types of errors in the future?
 Rachael Wilkenfeld
PowerScore Staff
  • PowerScore Staff
  • Posts: 1358
  • Joined: Dec 15, 2011
|
#93923
Hi cgs,

It sounds like you did a lot of things well in your initial attempt at this question. You recognized the problem with the argument. You found the conclusion to the argument. You saw how the conclusion did not lead to the conclusion drawn. All of these things are great great steps.

The best way to avoid an error like the one you made is that you could probably have avoided if you tried the assumption negation technique. If we negate answer choice (D), we'd have that there could be an additional explanation for the shrinking applicant pool. That doesn't mean that we have an additional explanation, just that it's possible. Even if there are additional explanations, that doesn't mean that raising tuition wouldn't solve the problem.

On the other hand, if the author is incorrect about the proposed reasoning, the conclusion wouldn't apply.

A better question for yourself is why you eliminated the correct answer choice. It's easy to get mislead by a wrong answer choice, but understanding why you didn't pick the right answer is key to improving your future performance. Did you eliminate answer choice (A) because you didn't leave it as a contender? Did you use the assumption negation technique on answer choice (A)? Think about those questions as you try to avoid these errors in the future.

Hope that helps!

Get the most out of your LSAT Prep Plus subscription.

Analyze and track your performance with our Testing and Analytics Package.